LSAT and Law School Admissions Forum

Get expert LSAT preparation and law school admissions advice from PowerScore Test Preparation.

 reop6780
  • Posts: 265
  • Joined: Jul 27, 2013
|
#14809
This is assumption question.

The correct answer is E.

While I was convinced with answer C, I figured it was wrong after negating it.

Even thought it is not the store's advantage to charge customers supposing a possible case that the store earns zero profit from charging customers for wrapping services, the conclusion can be solid with the disadvantages of free wrapping presented by the stimuli.

Then, I came up with similar idea that negated answer E would not harm the conclusion either.

What if ALL customers or NONE want the wrapping services?

It would be even more expensive to provide the service, or no advantage providing them.
(most-> all ; a few-> none)

How does the negated correct answer E damage the conclusion?
 Adam Tyson
PowerScore Staff
  • PowerScore Staff
  • Posts: 5153
  • Joined: Apr 14, 2011
|
#14811
With any assumption question, remember that the author has to believe the assumption is true in order to make his conclusion. That's where the assumption negation technique comes from - if you negate the assumption, you should either directly destroy the conclusion or else remove all support for it. The conclusion, in this case, is that there is no reason to offer gift wrapping. It looks to me like you knew that and went about testing your contenders in the right way. The problem came with the way you negated answer E.

A quick refresher on some concepts here involving numbers - "Some" means 1 all the way up to all; "None" means, of course, zero; "All" means, obviously, all; and "Not all" means anything from zero all the way up to one less than all. So where do "most" and "few" come in?

Most is mathematically provable - it means "more than half". That is a subset of "some" - it's more than half, and could go all the way up to all. Most folks wouldn't think that way intuitively, but if all the people reading this live on planet Earth, then it's true that most of the people reading this live on planet Earth. So, most already includes all.

Few is less certain than most, because it is entirely subjective. Few means a small number, but small is completely relative. "Few people on Earth are ambidextrous" might be a true statement, even if that number is in the millions - it's all about perspective and relativity. It's also true that "few people reading this right now live on the surface of the sun" - few INCLUDES none. An argument about few people wanting gift wrapping might include the idea of none wanting it. Certainly in this case, if none want it than there is no point to offering it, and the author's argument is strengthened. So, by choosing "all or none" as your negation, you weren't negating at all - you were taking "most" and "few" to their polar extremes.

To negate answer E more accurately, you would have to find some number that is MORE than few and LESS than most. That is, find something that fits neither condition. Since "few" is relative, it's hard to say what that number is, but what if it was exactly half? That probably fits the bill. If half the customers want it, then our author's argument falls apart - it's not most, so it's not necessarily too costly, and it's not "few," so there may indeed be an advantage to offering it. When we look at that third alternative (as if this was a False Dilemma problem) we see that there may actually be an advantage, directly contradicting the conclusion and satisfying the Assumption Negation Technique's test.

I hope that helped. You are clearly on the right track - keep it up!
 dorsa
  • Posts: 1
  • Joined: Aug 08, 2017
|
#38075
Hi
I don’t get why we can’t choose A. By Assumption Negation technique I can say that if costs less it was not that much important for them to do it according to pervious holiday season.
 Francis O'Rourke
PowerScore Staff
  • PowerScore Staff
  • Posts: 471
  • Joined: Mar 10, 2017
|
#38116
Hi Dorsa,

The logical negation of Choice (A) should read "Gift wrapping would not cost the store more this season than in previous season." This means that the cost is less or the same as in previous years. This fact would not attack the manager's conclusion, since the manager may still be right if the cost is the same or if the cost decreases only a little. Remember not to supply the polar opposite when applying the Assumption Negation Technique.
User avatar
 ashpine17
  • Posts: 321
  • Joined: Apr 06, 2021
|
#103150
Why c bad though is it because argument is talking about free and this is about charging?
 Rachael Wilkenfeld
PowerScore Staff
  • PowerScore Staff
  • Posts: 1358
  • Joined: Dec 15, 2011
|
#103381
Hi Ash,

Did you try the assumption negation technique here? If we negate answer choice (C), we get that it wouldn't be to the store's advantage to charge for wrapping services. Putting that back in the argument, we get that because it's not to the store's advantage to charge for wrapping services, there's no reason to offer free wrapping services. It doesn't weaken the conclusion that there's no reason to offer free wrapping services if it's not to their advantage to charge for wrapping services. It's somewhat irrelevant, right? If the negated form of an answer choice has no impact on the argument, it's not a necessary assumption.

Hope that helps!
User avatar
 yenisey
  • Posts: 19
  • Joined: Oct 14, 2021
|
#104963
Is this a "defender" or "supporter" assumption? I think it's neither. Are there many instances when it's neither supporter nor defender assumption? I didn't find a missing link, also the answer choices did not negate the possibility of a weakness for it to be a defender. Is there any other technique by which I can recognize the correct answer? I was stumped by this assumption question.
 Robert Carroll
PowerScore Staff
  • PowerScore Staff
  • Posts: 1787
  • Joined: Dec 06, 2013
|
#105033
yenisey,

My initial impression was actually that this answer was both! But thinking about it more, it's a Defender. The stimulus has a flaw. It's possible that the quantity of customers who want free gift-wrapping is somewhere in between "most" and "only a few." The manager didn't consider this possibility, which would ruin the argument. Therefore, the manager has to assume that this argument-destroying quantity isn't true, and that's what answer choice (E) does.

Every Assumption will indeed be a Supporter or Defender.

Robert Carroll

Get the most out of your LSAT Prep Plus subscription.

Analyze and track your performance with our Testing and Analytics Package.